WBR0990: Difference between revisions

Jump to navigation Jump to search
No edit summary
No edit summary
Line 1: Line 1:
{{WBRQuestion
{{WBRQuestion
|QuestionAuthor=William J Gibson
|QuestionAuthor=William J Gibson (Reviewed by Serge Korjian)
|ExamType=USMLE Step 1
|ExamType=USMLE Step 1
|MainCategory=Physiology
|MainCategory=Physiology
Line 21: Line 21:
|MainCategory=Physiology
|MainCategory=Physiology
|SubCategory=Pulmonology
|SubCategory=Pulmonology
|Prompt=An attending in the cardiac intensive care unit at major academic medical center is teaching a resident about the pulmonary vasculature.  A patient with a history of hypertrophic cardiomyopathy is recovering from an anterior myocardial infarction and the following values are measured.
|Prompt=An attending in the cardiac intensive care unit at a major academic medical center is teaching a resident about the pulmonary vasculature.  A patient with a history of hypertrophic cardiomyopathy is recovering from an anterior myocardial infarction and the following values below are measured. Which of the following corresponds to the pulmonary vascular resistance in this patient?
 
<br>
Pulmonary artery pressure (mean): 15
'''Pulmonary artery pressure (mean): 15
Pulmonary capillary wedge pressure (mean):  10
Pulmonary capillary wedge pressure (mean):  10
Aortic systolic pressure: 140
Aortic systolic pressure: 140
Aortic diastolic pressure: 90
Aortic diastolic pressure: 90
Heart rate: 90 per minute
Heart rate: 90 per minute
Stroke Volume: 40 mL
Stroke Volume: 40 mL'''
 


Which of the following corresponds to the pulmonary vascular resistance in this patient?
|Explanation=The pulmonary vascular resistance of a vessel can be calculated by the change in pressure across the segment divided by the flow (R=P/Q).  Generally, the pulmonary capillary wedge pressure is a good estimate of the pressure in the left atrium.  Therefore, pulmonary vascular resistance is given by the formula:
|Explanation=In this question we are asked to calculate the pulmonary vascular resistance for this patient.  Recall that the resistance of a vessel can be calculated by the change in pressure across the segment divided by the flow (R=P/Q).  Generally, the pulmonary capillary wedge pressure is a good estimate of the pressure in the left atrium.  Therefore, pulmonary vascular resistance is given by the formula:


[[File:PVREqn101814.png]]
[[File:PVREqn101814.png]]
Line 37: Line 37:
Plugging in values from the above case we get the following:
Plugging in values from the above case we get the following:
(15 mmHg - 10 mmHg)/(90 beats/minute*(.040 L/min))=5/3.60=1.39 mmHg·min/L.
(15 mmHg - 10 mmHg)/(90 beats/minute*(.040 L/min))=5/3.60=1.39 mmHg·min/L.
Normal values for pulmonary vascular resistance range from 0.25–1.6 mmHg·min/L.
Normal values for pulmonary vascular resistance range from 0.25–1.6 mmHg·min/L.
|AnswerA=0.72 mmHg·min/L
|AnswerA=0.72 mmHg·min/L

Revision as of 23:13, 15 August 2015

 
Author PageAuthor::William J Gibson (Reviewed by Serge Korjian)
Exam Type ExamType::USMLE Step 1
Main Category MainCategory::Physiology
Sub Category SubCategory::Pulmonology
Prompt [[Prompt::An attending in the cardiac intensive care unit at a major academic medical center is teaching a resident about the pulmonary vasculature. A patient with a history of hypertrophic cardiomyopathy is recovering from an anterior myocardial infarction and the following values below are measured. Which of the following corresponds to the pulmonary vascular resistance in this patient?


Pulmonary artery pressure (mean): 15 Pulmonary capillary wedge pressure (mean): 10 Aortic systolic pressure: 140 Aortic diastolic pressure: 90 Heart rate: 90 per minute Stroke Volume: 40 mL]]

Answer A AnswerA::0.72 mmHg·min/L
Answer A Explanation AnswerAExp::See calculation in the explanation
Answer B AnswerB::1.39 mmHg·min/L
Answer B Explanation AnswerBExp::See calculation in the explanation
Answer C AnswerC::11.25 mmHg·min/L
Answer C Explanation AnswerCExp::See calculation in the explanation
Answer D AnswerD::13.9 mmHg·min/L
Answer D Explanation AnswerDExp::This result can be achieved by dividing the difference in systolic and diastolic aortic pressures and dividing by the cardiac output (3.6L/min)
Answer E AnswerE::20.83 mmHg·min/L
Answer E Explanation AnswerEExp::See calculation in the explanation
Right Answer RightAnswer::B
Explanation [[Explanation::The pulmonary vascular resistance of a vessel can be calculated by the change in pressure across the segment divided by the flow (R=P/Q). Generally, the pulmonary capillary wedge pressure is a good estimate of the pressure in the left atrium. Therefore, pulmonary vascular resistance is given by the formula:

Plugging in values from the above case we get the following: (15 mmHg - 10 mmHg)/(90 beats/minute*(.040 L/min))=5/3.60=1.39 mmHg·min/L. Normal values for pulmonary vascular resistance range from 0.25–1.6 mmHg·min/L.
Educational Objective: Pulmonary vascular resistance is given by the change in pressure across the pulmonary vasculature divided by the cardiac output.
References: First Aid 2014 page 599]]

Approved Approved::Yes
Keyword WBRKeyword::Lung, WBRKeyword::Vasculature, WBRKeyword::Pulmonary hypertension, WBRKeyword::Hemodynamics, WBRKeyword::Hemodynamic
Linked Question Linked::
Order in Linked Questions LinkedOrder::